LSAT and Law School Admissions Forum

Get expert LSAT preparation and law school admissions advice from PowerScore Test Preparation.

User avatar
 Dave Killoran
PowerScore Staff
  • PowerScore Staff
  • Posts: 5852
  • Joined: Mar 25, 2011
|
#43591
Complete Question Explanation
(The complete setup for this game can be found here: lsat/viewtopic.php?t=1755)

The correct answer choice is (B)

The question stem invokes the “CCL Against” rule discussed in the setup. Accordingly, answer choice (B) is correct.
 darrowclaire@gmail.com
  • Posts: 1
  • Joined: Feb 23, 2023
|
#102082
Hi there,

I was wondering why E would not be the correct answer choice. I was able to determine the CCL would put us in the 4-3 distribution. Wouldn't that mean that both Moderates have to vote for Datalog? I thought that answer choice E puts us in the 5-2 distribution which cannot happen in the CCL scenario.

Thanks!
 Adam Tyson
PowerScore Staff
  • PowerScore Staff
  • Posts: 5153
  • Joined: Apr 14, 2011
|
#102086
The rules require at least one Conservative to vote Against Datalog, darrowclaire, so if they both vote the same way as each other, they must both be voting Against. Splitting the Liberals means at least one votes with those Conservatives, putting us into the CCL scenario in the Against group, so the two Moderates must also vote Against, leaving the other two Liberals to vote For. The outcome must be:

F: LL
A: CCLMM

Answer E Must Be True, which makes it a wrong answer. We need the one thing that Cannot Be True, and that's answer B. The Moderates cannot vote For, they must vote Against.

Get the most out of your LSAT Prep Plus subscription.

Analyze and track your performance with our Testing and Analytics Package.